LSAT and Law School Admissions Forum

Get expert LSAT preparation and law school admissions advice from PowerScore Test Preparation.

 Administrator
PowerScore Staff
  • PowerScore Staff
  • Posts: 8916
  • Joined: Feb 02, 2011
|
#81368
Complete Question Explanation

The correct answer choice is (E).

Answer choice (A):

Answer choice (B):

Answer choice (C):

Answer choice (D):

Answer choice (E): This is the correct answer choice.

This explanation is still in progress. Please post any questions below!
 avengingangel
  • Posts: 275
  • Joined: Jun 14, 2016
|
#30942
Hello! I did end up choosing the correct answer here, too (double woo!), but I have more of a general question about this problem. After reading the question stem, I wasn't sure if I would be looking for an answer choice that provides conclusive or inconclusive proof. As in, I wasn't sure if I would be looking for proof that supported his hypothesis or called it into question. In fact, I originally thought D was the right answer until I read E. Does that make sense ?? I don't know if I am to take "proof" for what the word means, or for how it was being used in the context of the sentence in the passage?

In other words, the answer choice I might have expected to be correct would be what's in (E), adding:...", although a small number of similar pollen fossils have been found in the uplands as well." (this example would be taking "Proof" as its being used in the context of the passage-inconclusive)

Thanks!
 Adam Tyson
PowerScore Staff
  • PowerScore Staff
  • Posts: 5153
  • Joined: Apr 14, 2011
|
#31317
If I understand what you are asking here, Angel, you want to know if we want to help prove that Haffer was right or help prove that he was wrong? Or are you asking if we want to add more inconclusive data to the mix?

Regardless of which direction you were taking, I'll try to point you in the right one now. The author refers us to the line that says "Although no conclusive proof has yet been found to support Haffer's hypothesis..." and asks us to contribute to that proof. What they are asking for is help supporting Haffer - something more conclusive than what we already have. That's what makes E a winner here - the pollen helps show that the lowlands had no rain forest during the ice age, supporting Haffer's claim that it was the diversity of the climate, and not its stability, that helped foster species diversity. That may not be conclusive, but it sure does help!

Adding what you suggested to answer E would make things less conclusive, and that's the opposite of what we want here. We want proof supporting Haffer, not more confusion over the issue. That would be a very weird LSAT question!

Keep at it, Angel!

Get the most out of your LSAT Prep Plus subscription.

Analyze and track your performance with our Testing and Analytics Package.